miniaufgabe.js ==== 3. Februar 2020 bis 7. Februar 2020 ==== === Montag 3. Februar 2020 === Keine Miniaufgaben. Für die späte Bekanntmachung entschuldige ich mich. === Donnerstag 6. Februar 2020 === Gegeben sind zwei Würfel $X$ und $Y$ (mit 6 Flächen) auf denen unterschiedliche Zahlen notiert sind. Berechnen Sie $P(X>Y)$ für folgende Augenzahlen auf den jeweiligen Würfeln:miniAufgabe("#exowuerfelkampf","#solwuerfelkampf", [["Würfel $X$: (2, 3, 3, 3, 3, 5), Würfel $Y$: (1, 1, 4, 4, 4, 5)", "\\[\\begin{align*}P(X>Y) = & P(X=2 \\text{ und } Y<2) + P(X=3 \\text{ und } Y<3) + P(X=5 \\text{ und } Y<5) = \\\\ & P(X=2)\\cdot P(Y<2) + P(X=3)\\cdot P(Y<3) + P(X=5)\\cdot P(Y<5) = \\\\ & \\frac{1}{6} \\cdot \\frac{2}{6} + \\frac{4}{6} \\cdot \\frac{2}{6} + \\frac{1}{6} \\cdot \\frac{5}{6} = \\frac{5}{12}\\end{align*}\\]"], ["Würfel $X$: (1, 1, 1, 2, 2, 3), Würfel $Y$: (1, 1, 2, 6, 6, 6)", "\\[\\begin{align*}P(X>Y) = & P(X=1 \\text{ und } Y<1) + P(X=2 \\text{ und } Y<2) + P(X=3 \\text{ und } Y<3) = \\\\ & P(X=1)\\cdot P(Y<1) + P(X=2)\\cdot P(Y<2) + P(X=3)\\cdot P(Y<3) = \\\\ & \\frac{3}{6} \\cdot \\frac{0}{6} + \\frac{2}{6} \\cdot \\frac{2}{6} + \\frac{1}{6} \\cdot \\frac{3}{6} = \\frac{7}{36}\\end{align*}\\]"], ["Würfel $X$: (3, 3, 3, 5, 5, 6), Würfel $Y$: (4, 4, 5, 5, 5, 6)", "\\[\\begin{align*}P(X>Y) = & P(X=3 \\text{ und } Y<3) + P(X=5 \\text{ und } Y<5) + P(X=6 \\text{ und } Y<6) = \\\\ & P(X=3)\\cdot P(Y<3) + P(X=5)\\cdot P(Y<5) + P(X=6)\\cdot P(Y<6) = \\\\ & \\frac{3}{6} \\cdot \\frac{0}{6} + \\frac{2}{6} \\cdot \\frac{2}{6} + \\frac{1}{6} \\cdot \\frac{5}{6} = \\frac{1}{4}\\end{align*}\\]"], ["Würfel $X$: (2, 2, 2, 5, 6, 6), Würfel $Y$: (3, 3, 3, 5, 6, 6)", "\\[\\begin{align*}P(X>Y) = & P(X=2 \\text{ und } Y<2) + P(X=5 \\text{ und } Y<5) + P(X=6 \\text{ und } Y<6) = \\\\ & P(X=2)\\cdot P(Y<2) + P(X=5)\\cdot P(Y<5) + P(X=6)\\cdot P(Y<6) = \\\\ & \\frac{3}{6} \\cdot \\frac{0}{6} + \\frac{1}{6} \\cdot \\frac{3}{6} + \\frac{2}{6} \\cdot \\frac{4}{6} = \\frac{11}{36}\\end{align*}\\]"], ["Würfel $X$: (3, 4, 4, 4, 5, 5), Würfel $Y$: (3, 3, 5, 5, 6, 6)", "\\[\\begin{align*}P(X>Y) = & P(X=3 \\text{ und } Y<3) + P(X=4 \\text{ und } Y<4) + P(X=5 \\text{ und } Y<5) = \\\\ & P(X=3)\\cdot P(Y<3) + P(X=4)\\cdot P(Y<4) + P(X=5)\\cdot P(Y<5) = \\\\ & \\frac{1}{6} \\cdot \\frac{0}{6} + \\frac{3}{6} \\cdot \\frac{2}{6} + \\frac{2}{6} \\cdot \\frac{2}{6} = \\frac{5}{18}\\end{align*}\\]"], ["Würfel $X$: (2, 2, 2, 4, 5, 5), Würfel $Y$: (3, 4, 4, 6, 6, 6)", "\\[\\begin{align*}P(X>Y) = & P(X=2 \\text{ und } Y<2) + P(X=4 \\text{ und } Y<4) + P(X=5 \\text{ und } Y<5) = \\\\ & P(X=2)\\cdot P(Y<2) + P(X=4)\\cdot P(Y<4) + P(X=5)\\cdot P(Y<5) = \\\\ & \\frac{3}{6} \\cdot \\frac{0}{6} + \\frac{1}{6} \\cdot \\frac{1}{6} + \\frac{2}{6} \\cdot \\frac{3}{6} = \\frac{7}{36}\\end{align*}\\]"], ["Würfel $X$: (2, 4, 4, 6, 6, 6), Würfel $Y$: (1, 1, 2, 2, 5, 5)", "\\[\\begin{align*}P(X>Y) = & P(X=2 \\text{ und } Y<2) + P(X=4 \\text{ und } Y<4) + P(X=6 \\text{ und } Y<6) = \\\\ & P(X=2)\\cdot P(Y<2) + P(X=4)\\cdot P(Y<4) + P(X=6)\\cdot P(Y<6) = \\\\ & \\frac{1}{6} \\cdot \\frac{2}{6} + \\frac{2}{6} \\cdot \\frac{4}{6} + \\frac{3}{6} \\cdot \\frac{6}{6} = \\frac{7}{9}\\end{align*}\\]"], ["Würfel $X$: (4, 5, 5, 5, 6, 6), Würfel $Y$: (1, 1, 1, 4, 6, 6)", "\\[\\begin{align*}P(X>Y) = & P(X=4 \\text{ und } Y<4) + P(X=5 \\text{ und } Y<5) + P(X=6 \\text{ und } Y<6) = \\\\ & P(X=4)\\cdot P(Y<4) + P(X=5)\\cdot P(Y<5) + P(X=6)\\cdot P(Y<6) = \\\\ & \\frac{1}{6} \\cdot \\frac{3}{6} + \\frac{3}{6} \\cdot \\frac{4}{6} + \\frac{2}{6} \\cdot \\frac{4}{6} = \\frac{23}{36}\\end{align*}\\]"], ["Würfel $X$: (2, 2, 3, 5, 5, 5), Würfel $Y$: (1, 1, 2, 2, 4, 4)", "\\[\\begin{align*}P(X>Y) = & P(X=2 \\text{ und } Y<2) + P(X=3 \\text{ und } Y<3) + P(X=5 \\text{ und } Y<5) = \\\\ & P(X=2)\\cdot P(Y<2) + P(X=3)\\cdot P(Y<3) + P(X=5)\\cdot P(Y<5) = \\\\ & \\frac{2}{6} \\cdot \\frac{2}{6} + \\frac{1}{6} \\cdot \\frac{4}{6} + \\frac{3}{6} \\cdot \\frac{6}{6} = \\frac{13}{18}\\end{align*}\\]"], ["Würfel $X$: (2, 2, 3, 3, 6, 6), Würfel $Y$: (1, 3, 3, 6, 6, 6)", "\\[\\begin{align*}P(X>Y) = & P(X=2 \\text{ und } Y<2) + P(X=3 \\text{ und } Y<3) + P(X=6 \\text{ und } Y<6) = \\\\ & P(X=2)\\cdot P(Y<2) + P(X=3)\\cdot P(Y<3) + P(X=6)\\cdot P(Y<6) = \\\\ & \\frac{2}{6} \\cdot \\frac{1}{6} + \\frac{2}{6} \\cdot \\frac{1}{6} + \\frac{2}{6} \\cdot \\frac{3}{6} = \\frac{5}{18}\\end{align*}\\]"]], "
");